range of $3x^2-2xy$ subjected to $x^2+y^2=1$












2












$begingroup$


If $x^2+y^2=1$. then the range of expression $3x^2-2xy$ without trigonometric substitution method



what i have done try here is use arithmetic geometric inequality



$displaystyle x^2+y^2geq 2xy$



$displaystyle -2xygeq -(x^2+y^2)$



$displaystyle 3x^2-2xygeq 2x^2-y^2$



this will not help more



how do i solve it help me please










share|cite|improve this question











$endgroup$












  • $begingroup$
    Maybe put $x=cos t, y=sin t$ and plug these into objective function, then constraint automatic and need to get range of resulting trig function.
    $endgroup$
    – coffeemath
    Dec 17 '18 at 18:34










  • $begingroup$
    "without trigonometric substitution"
    $endgroup$
    – David G. Stork
    Dec 17 '18 at 18:39
















2












$begingroup$


If $x^2+y^2=1$. then the range of expression $3x^2-2xy$ without trigonometric substitution method



what i have done try here is use arithmetic geometric inequality



$displaystyle x^2+y^2geq 2xy$



$displaystyle -2xygeq -(x^2+y^2)$



$displaystyle 3x^2-2xygeq 2x^2-y^2$



this will not help more



how do i solve it help me please










share|cite|improve this question











$endgroup$












  • $begingroup$
    Maybe put $x=cos t, y=sin t$ and plug these into objective function, then constraint automatic and need to get range of resulting trig function.
    $endgroup$
    – coffeemath
    Dec 17 '18 at 18:34










  • $begingroup$
    "without trigonometric substitution"
    $endgroup$
    – David G. Stork
    Dec 17 '18 at 18:39














2












2








2





$begingroup$


If $x^2+y^2=1$. then the range of expression $3x^2-2xy$ without trigonometric substitution method



what i have done try here is use arithmetic geometric inequality



$displaystyle x^2+y^2geq 2xy$



$displaystyle -2xygeq -(x^2+y^2)$



$displaystyle 3x^2-2xygeq 2x^2-y^2$



this will not help more



how do i solve it help me please










share|cite|improve this question











$endgroup$




If $x^2+y^2=1$. then the range of expression $3x^2-2xy$ without trigonometric substitution method



what i have done try here is use arithmetic geometric inequality



$displaystyle x^2+y^2geq 2xy$



$displaystyle -2xygeq -(x^2+y^2)$



$displaystyle 3x^2-2xygeq 2x^2-y^2$



this will not help more



how do i solve it help me please







inequality






share|cite|improve this question















share|cite|improve this question













share|cite|improve this question




share|cite|improve this question








edited Dec 17 '18 at 18:31







jacky

















asked Dec 17 '18 at 18:28









jackyjacky

1,235815




1,235815












  • $begingroup$
    Maybe put $x=cos t, y=sin t$ and plug these into objective function, then constraint automatic and need to get range of resulting trig function.
    $endgroup$
    – coffeemath
    Dec 17 '18 at 18:34










  • $begingroup$
    "without trigonometric substitution"
    $endgroup$
    – David G. Stork
    Dec 17 '18 at 18:39


















  • $begingroup$
    Maybe put $x=cos t, y=sin t$ and plug these into objective function, then constraint automatic and need to get range of resulting trig function.
    $endgroup$
    – coffeemath
    Dec 17 '18 at 18:34










  • $begingroup$
    "without trigonometric substitution"
    $endgroup$
    – David G. Stork
    Dec 17 '18 at 18:39
















$begingroup$
Maybe put $x=cos t, y=sin t$ and plug these into objective function, then constraint automatic and need to get range of resulting trig function.
$endgroup$
– coffeemath
Dec 17 '18 at 18:34




$begingroup$
Maybe put $x=cos t, y=sin t$ and plug these into objective function, then constraint automatic and need to get range of resulting trig function.
$endgroup$
– coffeemath
Dec 17 '18 at 18:34












$begingroup$
"without trigonometric substitution"
$endgroup$
– David G. Stork
Dec 17 '18 at 18:39




$begingroup$
"without trigonometric substitution"
$endgroup$
– David G. Stork
Dec 17 '18 at 18:39










6 Answers
6






active

oldest

votes


















2












$begingroup$

If you have the requisite linear algebra theorems at your disposal, then you can recognize



$$3x^2-2xy=pmatrix{x&y}pmatrix{3&-1\-1&0}pmatrix{x\y}$$



where the $2times2$ matrix is selfadjoint with eigenvalues satisfying $lambda^2-3lambda-1=0$. For selfadjoint matrices $M$, the corresponding eigenvectors are orthogonal, and thus when the vector $mathbf{x}$ ranges over the unit ball (in this case the unit circle), $mathbf{x}^TMmathbf{x}$ ranges from the smallest eigenvalue to the largest, in this case, from $(3-sqrt{13})/2$ to $(3+sqrt{13})/2$.






share|cite|improve this answer









$endgroup$





















    1












    $begingroup$

    Since $x^2+y^2=1$, you can substitute for $y=pmsqrt{1-x^2}$ in $f(x,y)=3x^2-2xy$ to get $g(x)=f(x,pmsqrt{1-x^2})=3x^2pm2xsqrt{1-x^2},-1le xle1$ and find the extrema using standard techniques.






    share|cite|improve this answer









    $endgroup$





















      1












      $begingroup$

      The range that you're after is $left[frac{3-sqrt{13}}2,frac{3+sqrt{13}}2right]$. In fact, if $x^2+y^2=1$, then$$3x^2-2xy=3x^2pm xsqrt{1-x^2}.$$So, for each $xin[-1,1]$, let$$f(x)=3x^2-2xsqrt{1-x^2}text{ and let }g(x)=3x^2+xsqrt{1-x^2}.$$Now, using that standard Calculus techniques, you can check that the range of both functions $f$ and $g$ is $left[frac{3-sqrt{13}}2,frac{3+sqrt{13}}2right]$.






      share|cite|improve this answer









      $endgroup$





















        1












        $begingroup$

        $3 x^2 - 2 x y$ with $y = pm sqrt{1-x^2}$ is $3 x^2 pm 2 x sqrt{1 - x^2}:$



        enter image description here



        Take derivative w.r.t. $x$, set it to zero and solve.



        Note that the two functions are reflections about $x=0$, so their ranges are the same.






        share|cite|improve this answer











        $endgroup$













        • $begingroup$
          Note $y$ could also be the negative of what you have.
          $endgroup$
          – coffeemath
          Dec 17 '18 at 18:35










        • $begingroup$
          It doesn't make a difference, the two functions are reflections of each other about $y$ axis and have the same range
          $endgroup$
          – Shubham Johri
          Dec 17 '18 at 18:40



















        1












        $begingroup$

        For a calculs-free approach, note that
        $$
        frac{3+sqrt{13}}{2}left[frac{2y-(3+sqrt{13})x}{sqrt{26+6sqrt{13}}}right]^2+frac{3-sqrt{13}}{2}left[frac{2y-(3-sqrt{13})x}{sqrt{26-6sqrt{13}}}right]^2=3x^2-2xy
        $$

        and
        $$
        left[frac{2y-(3+sqrt{13})x}{sqrt{26+6sqrt{13}}}right]^2+left[frac{2y-(3-sqrt{13})x}{sqrt{26-6sqrt{13}}}right]^2=x^2+y^2
        $$

        So the range of $3x^2-2xy$, as $(x,y)$ varies on the unit circle, is the closed interval with endpoints $dfrac{3pmsqrt{13}}{2}$.






        share|cite|improve this answer









        $endgroup$





















          1












          $begingroup$

          Making $y = lambda x$ and substituting we have



          $$
          mbox{Variation for} x^2(3-2lambda) mbox{s. t. } x^2(1+lambda^2) = 1
          $$



          or variation for



          $$
          f(lambda) = frac{3-2lambda}{1+lambda^2}
          $$



          now determining the stationary points with $f'(lambda) = 0$ obtaining



          $$
          lambda = frac 12left(3pmsqrt{13}right)
          $$



          etc.






          share|cite|improve this answer









          $endgroup$













            Your Answer





            StackExchange.ifUsing("editor", function () {
            return StackExchange.using("mathjaxEditing", function () {
            StackExchange.MarkdownEditor.creationCallbacks.add(function (editor, postfix) {
            StackExchange.mathjaxEditing.prepareWmdForMathJax(editor, postfix, [["$", "$"], ["\\(","\\)"]]);
            });
            });
            }, "mathjax-editing");

            StackExchange.ready(function() {
            var channelOptions = {
            tags: "".split(" "),
            id: "69"
            };
            initTagRenderer("".split(" "), "".split(" "), channelOptions);

            StackExchange.using("externalEditor", function() {
            // Have to fire editor after snippets, if snippets enabled
            if (StackExchange.settings.snippets.snippetsEnabled) {
            StackExchange.using("snippets", function() {
            createEditor();
            });
            }
            else {
            createEditor();
            }
            });

            function createEditor() {
            StackExchange.prepareEditor({
            heartbeatType: 'answer',
            autoActivateHeartbeat: false,
            convertImagesToLinks: true,
            noModals: true,
            showLowRepImageUploadWarning: true,
            reputationToPostImages: 10,
            bindNavPrevention: true,
            postfix: "",
            imageUploader: {
            brandingHtml: "Powered by u003ca class="icon-imgur-white" href="https://imgur.com/"u003eu003c/au003e",
            contentPolicyHtml: "User contributions licensed under u003ca href="https://creativecommons.org/licenses/by-sa/3.0/"u003ecc by-sa 3.0 with attribution requiredu003c/au003e u003ca href="https://stackoverflow.com/legal/content-policy"u003e(content policy)u003c/au003e",
            allowUrls: true
            },
            noCode: true, onDemand: true,
            discardSelector: ".discard-answer"
            ,immediatelyShowMarkdownHelp:true
            });


            }
            });














            draft saved

            draft discarded


















            StackExchange.ready(
            function () {
            StackExchange.openid.initPostLogin('.new-post-login', 'https%3a%2f%2fmath.stackexchange.com%2fquestions%2f3044271%2frange-of-3x2-2xy-subjected-to-x2y2-1%23new-answer', 'question_page');
            }
            );

            Post as a guest















            Required, but never shown

























            6 Answers
            6






            active

            oldest

            votes








            6 Answers
            6






            active

            oldest

            votes









            active

            oldest

            votes






            active

            oldest

            votes









            2












            $begingroup$

            If you have the requisite linear algebra theorems at your disposal, then you can recognize



            $$3x^2-2xy=pmatrix{x&y}pmatrix{3&-1\-1&0}pmatrix{x\y}$$



            where the $2times2$ matrix is selfadjoint with eigenvalues satisfying $lambda^2-3lambda-1=0$. For selfadjoint matrices $M$, the corresponding eigenvectors are orthogonal, and thus when the vector $mathbf{x}$ ranges over the unit ball (in this case the unit circle), $mathbf{x}^TMmathbf{x}$ ranges from the smallest eigenvalue to the largest, in this case, from $(3-sqrt{13})/2$ to $(3+sqrt{13})/2$.






            share|cite|improve this answer









            $endgroup$


















              2












              $begingroup$

              If you have the requisite linear algebra theorems at your disposal, then you can recognize



              $$3x^2-2xy=pmatrix{x&y}pmatrix{3&-1\-1&0}pmatrix{x\y}$$



              where the $2times2$ matrix is selfadjoint with eigenvalues satisfying $lambda^2-3lambda-1=0$. For selfadjoint matrices $M$, the corresponding eigenvectors are orthogonal, and thus when the vector $mathbf{x}$ ranges over the unit ball (in this case the unit circle), $mathbf{x}^TMmathbf{x}$ ranges from the smallest eigenvalue to the largest, in this case, from $(3-sqrt{13})/2$ to $(3+sqrt{13})/2$.






              share|cite|improve this answer









              $endgroup$
















                2












                2








                2





                $begingroup$

                If you have the requisite linear algebra theorems at your disposal, then you can recognize



                $$3x^2-2xy=pmatrix{x&y}pmatrix{3&-1\-1&0}pmatrix{x\y}$$



                where the $2times2$ matrix is selfadjoint with eigenvalues satisfying $lambda^2-3lambda-1=0$. For selfadjoint matrices $M$, the corresponding eigenvectors are orthogonal, and thus when the vector $mathbf{x}$ ranges over the unit ball (in this case the unit circle), $mathbf{x}^TMmathbf{x}$ ranges from the smallest eigenvalue to the largest, in this case, from $(3-sqrt{13})/2$ to $(3+sqrt{13})/2$.






                share|cite|improve this answer









                $endgroup$



                If you have the requisite linear algebra theorems at your disposal, then you can recognize



                $$3x^2-2xy=pmatrix{x&y}pmatrix{3&-1\-1&0}pmatrix{x\y}$$



                where the $2times2$ matrix is selfadjoint with eigenvalues satisfying $lambda^2-3lambda-1=0$. For selfadjoint matrices $M$, the corresponding eigenvectors are orthogonal, and thus when the vector $mathbf{x}$ ranges over the unit ball (in this case the unit circle), $mathbf{x}^TMmathbf{x}$ ranges from the smallest eigenvalue to the largest, in this case, from $(3-sqrt{13})/2$ to $(3+sqrt{13})/2$.







                share|cite|improve this answer












                share|cite|improve this answer



                share|cite|improve this answer










                answered Dec 17 '18 at 19:13









                Barry CipraBarry Cipra

                60.3k654127




                60.3k654127























                    1












                    $begingroup$

                    Since $x^2+y^2=1$, you can substitute for $y=pmsqrt{1-x^2}$ in $f(x,y)=3x^2-2xy$ to get $g(x)=f(x,pmsqrt{1-x^2})=3x^2pm2xsqrt{1-x^2},-1le xle1$ and find the extrema using standard techniques.






                    share|cite|improve this answer









                    $endgroup$


















                      1












                      $begingroup$

                      Since $x^2+y^2=1$, you can substitute for $y=pmsqrt{1-x^2}$ in $f(x,y)=3x^2-2xy$ to get $g(x)=f(x,pmsqrt{1-x^2})=3x^2pm2xsqrt{1-x^2},-1le xle1$ and find the extrema using standard techniques.






                      share|cite|improve this answer









                      $endgroup$
















                        1












                        1








                        1





                        $begingroup$

                        Since $x^2+y^2=1$, you can substitute for $y=pmsqrt{1-x^2}$ in $f(x,y)=3x^2-2xy$ to get $g(x)=f(x,pmsqrt{1-x^2})=3x^2pm2xsqrt{1-x^2},-1le xle1$ and find the extrema using standard techniques.






                        share|cite|improve this answer









                        $endgroup$



                        Since $x^2+y^2=1$, you can substitute for $y=pmsqrt{1-x^2}$ in $f(x,y)=3x^2-2xy$ to get $g(x)=f(x,pmsqrt{1-x^2})=3x^2pm2xsqrt{1-x^2},-1le xle1$ and find the extrema using standard techniques.







                        share|cite|improve this answer












                        share|cite|improve this answer



                        share|cite|improve this answer










                        answered Dec 17 '18 at 18:35









                        Shubham JohriShubham Johri

                        5,337718




                        5,337718























                            1












                            $begingroup$

                            The range that you're after is $left[frac{3-sqrt{13}}2,frac{3+sqrt{13}}2right]$. In fact, if $x^2+y^2=1$, then$$3x^2-2xy=3x^2pm xsqrt{1-x^2}.$$So, for each $xin[-1,1]$, let$$f(x)=3x^2-2xsqrt{1-x^2}text{ and let }g(x)=3x^2+xsqrt{1-x^2}.$$Now, using that standard Calculus techniques, you can check that the range of both functions $f$ and $g$ is $left[frac{3-sqrt{13}}2,frac{3+sqrt{13}}2right]$.






                            share|cite|improve this answer









                            $endgroup$


















                              1












                              $begingroup$

                              The range that you're after is $left[frac{3-sqrt{13}}2,frac{3+sqrt{13}}2right]$. In fact, if $x^2+y^2=1$, then$$3x^2-2xy=3x^2pm xsqrt{1-x^2}.$$So, for each $xin[-1,1]$, let$$f(x)=3x^2-2xsqrt{1-x^2}text{ and let }g(x)=3x^2+xsqrt{1-x^2}.$$Now, using that standard Calculus techniques, you can check that the range of both functions $f$ and $g$ is $left[frac{3-sqrt{13}}2,frac{3+sqrt{13}}2right]$.






                              share|cite|improve this answer









                              $endgroup$
















                                1












                                1








                                1





                                $begingroup$

                                The range that you're after is $left[frac{3-sqrt{13}}2,frac{3+sqrt{13}}2right]$. In fact, if $x^2+y^2=1$, then$$3x^2-2xy=3x^2pm xsqrt{1-x^2}.$$So, for each $xin[-1,1]$, let$$f(x)=3x^2-2xsqrt{1-x^2}text{ and let }g(x)=3x^2+xsqrt{1-x^2}.$$Now, using that standard Calculus techniques, you can check that the range of both functions $f$ and $g$ is $left[frac{3-sqrt{13}}2,frac{3+sqrt{13}}2right]$.






                                share|cite|improve this answer









                                $endgroup$



                                The range that you're after is $left[frac{3-sqrt{13}}2,frac{3+sqrt{13}}2right]$. In fact, if $x^2+y^2=1$, then$$3x^2-2xy=3x^2pm xsqrt{1-x^2}.$$So, for each $xin[-1,1]$, let$$f(x)=3x^2-2xsqrt{1-x^2}text{ and let }g(x)=3x^2+xsqrt{1-x^2}.$$Now, using that standard Calculus techniques, you can check that the range of both functions $f$ and $g$ is $left[frac{3-sqrt{13}}2,frac{3+sqrt{13}}2right]$.







                                share|cite|improve this answer












                                share|cite|improve this answer



                                share|cite|improve this answer










                                answered Dec 17 '18 at 18:36









                                José Carlos SantosJosé Carlos Santos

                                169k23132237




                                169k23132237























                                    1












                                    $begingroup$

                                    $3 x^2 - 2 x y$ with $y = pm sqrt{1-x^2}$ is $3 x^2 pm 2 x sqrt{1 - x^2}:$



                                    enter image description here



                                    Take derivative w.r.t. $x$, set it to zero and solve.



                                    Note that the two functions are reflections about $x=0$, so their ranges are the same.






                                    share|cite|improve this answer











                                    $endgroup$













                                    • $begingroup$
                                      Note $y$ could also be the negative of what you have.
                                      $endgroup$
                                      – coffeemath
                                      Dec 17 '18 at 18:35










                                    • $begingroup$
                                      It doesn't make a difference, the two functions are reflections of each other about $y$ axis and have the same range
                                      $endgroup$
                                      – Shubham Johri
                                      Dec 17 '18 at 18:40
















                                    1












                                    $begingroup$

                                    $3 x^2 - 2 x y$ with $y = pm sqrt{1-x^2}$ is $3 x^2 pm 2 x sqrt{1 - x^2}:$



                                    enter image description here



                                    Take derivative w.r.t. $x$, set it to zero and solve.



                                    Note that the two functions are reflections about $x=0$, so their ranges are the same.






                                    share|cite|improve this answer











                                    $endgroup$













                                    • $begingroup$
                                      Note $y$ could also be the negative of what you have.
                                      $endgroup$
                                      – coffeemath
                                      Dec 17 '18 at 18:35










                                    • $begingroup$
                                      It doesn't make a difference, the two functions are reflections of each other about $y$ axis and have the same range
                                      $endgroup$
                                      – Shubham Johri
                                      Dec 17 '18 at 18:40














                                    1












                                    1








                                    1





                                    $begingroup$

                                    $3 x^2 - 2 x y$ with $y = pm sqrt{1-x^2}$ is $3 x^2 pm 2 x sqrt{1 - x^2}:$



                                    enter image description here



                                    Take derivative w.r.t. $x$, set it to zero and solve.



                                    Note that the two functions are reflections about $x=0$, so their ranges are the same.






                                    share|cite|improve this answer











                                    $endgroup$



                                    $3 x^2 - 2 x y$ with $y = pm sqrt{1-x^2}$ is $3 x^2 pm 2 x sqrt{1 - x^2}:$



                                    enter image description here



                                    Take derivative w.r.t. $x$, set it to zero and solve.



                                    Note that the two functions are reflections about $x=0$, so their ranges are the same.







                                    share|cite|improve this answer














                                    share|cite|improve this answer



                                    share|cite|improve this answer








                                    edited Dec 17 '18 at 18:44

























                                    answered Dec 17 '18 at 18:34









                                    David G. StorkDavid G. Stork

                                    11.1k41432




                                    11.1k41432












                                    • $begingroup$
                                      Note $y$ could also be the negative of what you have.
                                      $endgroup$
                                      – coffeemath
                                      Dec 17 '18 at 18:35










                                    • $begingroup$
                                      It doesn't make a difference, the two functions are reflections of each other about $y$ axis and have the same range
                                      $endgroup$
                                      – Shubham Johri
                                      Dec 17 '18 at 18:40


















                                    • $begingroup$
                                      Note $y$ could also be the negative of what you have.
                                      $endgroup$
                                      – coffeemath
                                      Dec 17 '18 at 18:35










                                    • $begingroup$
                                      It doesn't make a difference, the two functions are reflections of each other about $y$ axis and have the same range
                                      $endgroup$
                                      – Shubham Johri
                                      Dec 17 '18 at 18:40
















                                    $begingroup$
                                    Note $y$ could also be the negative of what you have.
                                    $endgroup$
                                    – coffeemath
                                    Dec 17 '18 at 18:35




                                    $begingroup$
                                    Note $y$ could also be the negative of what you have.
                                    $endgroup$
                                    – coffeemath
                                    Dec 17 '18 at 18:35












                                    $begingroup$
                                    It doesn't make a difference, the two functions are reflections of each other about $y$ axis and have the same range
                                    $endgroup$
                                    – Shubham Johri
                                    Dec 17 '18 at 18:40




                                    $begingroup$
                                    It doesn't make a difference, the two functions are reflections of each other about $y$ axis and have the same range
                                    $endgroup$
                                    – Shubham Johri
                                    Dec 17 '18 at 18:40











                                    1












                                    $begingroup$

                                    For a calculs-free approach, note that
                                    $$
                                    frac{3+sqrt{13}}{2}left[frac{2y-(3+sqrt{13})x}{sqrt{26+6sqrt{13}}}right]^2+frac{3-sqrt{13}}{2}left[frac{2y-(3-sqrt{13})x}{sqrt{26-6sqrt{13}}}right]^2=3x^2-2xy
                                    $$

                                    and
                                    $$
                                    left[frac{2y-(3+sqrt{13})x}{sqrt{26+6sqrt{13}}}right]^2+left[frac{2y-(3-sqrt{13})x}{sqrt{26-6sqrt{13}}}right]^2=x^2+y^2
                                    $$

                                    So the range of $3x^2-2xy$, as $(x,y)$ varies on the unit circle, is the closed interval with endpoints $dfrac{3pmsqrt{13}}{2}$.






                                    share|cite|improve this answer









                                    $endgroup$


















                                      1












                                      $begingroup$

                                      For a calculs-free approach, note that
                                      $$
                                      frac{3+sqrt{13}}{2}left[frac{2y-(3+sqrt{13})x}{sqrt{26+6sqrt{13}}}right]^2+frac{3-sqrt{13}}{2}left[frac{2y-(3-sqrt{13})x}{sqrt{26-6sqrt{13}}}right]^2=3x^2-2xy
                                      $$

                                      and
                                      $$
                                      left[frac{2y-(3+sqrt{13})x}{sqrt{26+6sqrt{13}}}right]^2+left[frac{2y-(3-sqrt{13})x}{sqrt{26-6sqrt{13}}}right]^2=x^2+y^2
                                      $$

                                      So the range of $3x^2-2xy$, as $(x,y)$ varies on the unit circle, is the closed interval with endpoints $dfrac{3pmsqrt{13}}{2}$.






                                      share|cite|improve this answer









                                      $endgroup$
















                                        1












                                        1








                                        1





                                        $begingroup$

                                        For a calculs-free approach, note that
                                        $$
                                        frac{3+sqrt{13}}{2}left[frac{2y-(3+sqrt{13})x}{sqrt{26+6sqrt{13}}}right]^2+frac{3-sqrt{13}}{2}left[frac{2y-(3-sqrt{13})x}{sqrt{26-6sqrt{13}}}right]^2=3x^2-2xy
                                        $$

                                        and
                                        $$
                                        left[frac{2y-(3+sqrt{13})x}{sqrt{26+6sqrt{13}}}right]^2+left[frac{2y-(3-sqrt{13})x}{sqrt{26-6sqrt{13}}}right]^2=x^2+y^2
                                        $$

                                        So the range of $3x^2-2xy$, as $(x,y)$ varies on the unit circle, is the closed interval with endpoints $dfrac{3pmsqrt{13}}{2}$.






                                        share|cite|improve this answer









                                        $endgroup$



                                        For a calculs-free approach, note that
                                        $$
                                        frac{3+sqrt{13}}{2}left[frac{2y-(3+sqrt{13})x}{sqrt{26+6sqrt{13}}}right]^2+frac{3-sqrt{13}}{2}left[frac{2y-(3-sqrt{13})x}{sqrt{26-6sqrt{13}}}right]^2=3x^2-2xy
                                        $$

                                        and
                                        $$
                                        left[frac{2y-(3+sqrt{13})x}{sqrt{26+6sqrt{13}}}right]^2+left[frac{2y-(3-sqrt{13})x}{sqrt{26-6sqrt{13}}}right]^2=x^2+y^2
                                        $$

                                        So the range of $3x^2-2xy$, as $(x,y)$ varies on the unit circle, is the closed interval with endpoints $dfrac{3pmsqrt{13}}{2}$.







                                        share|cite|improve this answer












                                        share|cite|improve this answer



                                        share|cite|improve this answer










                                        answered Dec 17 '18 at 18:49









                                        user10354138user10354138

                                        7,4322925




                                        7,4322925























                                            1












                                            $begingroup$

                                            Making $y = lambda x$ and substituting we have



                                            $$
                                            mbox{Variation for} x^2(3-2lambda) mbox{s. t. } x^2(1+lambda^2) = 1
                                            $$



                                            or variation for



                                            $$
                                            f(lambda) = frac{3-2lambda}{1+lambda^2}
                                            $$



                                            now determining the stationary points with $f'(lambda) = 0$ obtaining



                                            $$
                                            lambda = frac 12left(3pmsqrt{13}right)
                                            $$



                                            etc.






                                            share|cite|improve this answer









                                            $endgroup$


















                                              1












                                              $begingroup$

                                              Making $y = lambda x$ and substituting we have



                                              $$
                                              mbox{Variation for} x^2(3-2lambda) mbox{s. t. } x^2(1+lambda^2) = 1
                                              $$



                                              or variation for



                                              $$
                                              f(lambda) = frac{3-2lambda}{1+lambda^2}
                                              $$



                                              now determining the stationary points with $f'(lambda) = 0$ obtaining



                                              $$
                                              lambda = frac 12left(3pmsqrt{13}right)
                                              $$



                                              etc.






                                              share|cite|improve this answer









                                              $endgroup$
















                                                1












                                                1








                                                1





                                                $begingroup$

                                                Making $y = lambda x$ and substituting we have



                                                $$
                                                mbox{Variation for} x^2(3-2lambda) mbox{s. t. } x^2(1+lambda^2) = 1
                                                $$



                                                or variation for



                                                $$
                                                f(lambda) = frac{3-2lambda}{1+lambda^2}
                                                $$



                                                now determining the stationary points with $f'(lambda) = 0$ obtaining



                                                $$
                                                lambda = frac 12left(3pmsqrt{13}right)
                                                $$



                                                etc.






                                                share|cite|improve this answer









                                                $endgroup$



                                                Making $y = lambda x$ and substituting we have



                                                $$
                                                mbox{Variation for} x^2(3-2lambda) mbox{s. t. } x^2(1+lambda^2) = 1
                                                $$



                                                or variation for



                                                $$
                                                f(lambda) = frac{3-2lambda}{1+lambda^2}
                                                $$



                                                now determining the stationary points with $f'(lambda) = 0$ obtaining



                                                $$
                                                lambda = frac 12left(3pmsqrt{13}right)
                                                $$



                                                etc.







                                                share|cite|improve this answer












                                                share|cite|improve this answer



                                                share|cite|improve this answer










                                                answered Dec 17 '18 at 20:44









                                                CesareoCesareo

                                                9,4563517




                                                9,4563517






























                                                    draft saved

                                                    draft discarded




















































                                                    Thanks for contributing an answer to Mathematics Stack Exchange!


                                                    • Please be sure to answer the question. Provide details and share your research!

                                                    But avoid



                                                    • Asking for help, clarification, or responding to other answers.

                                                    • Making statements based on opinion; back them up with references or personal experience.


                                                    Use MathJax to format equations. MathJax reference.


                                                    To learn more, see our tips on writing great answers.




                                                    draft saved


                                                    draft discarded














                                                    StackExchange.ready(
                                                    function () {
                                                    StackExchange.openid.initPostLogin('.new-post-login', 'https%3a%2f%2fmath.stackexchange.com%2fquestions%2f3044271%2frange-of-3x2-2xy-subjected-to-x2y2-1%23new-answer', 'question_page');
                                                    }
                                                    );

                                                    Post as a guest















                                                    Required, but never shown





















































                                                    Required, but never shown














                                                    Required, but never shown












                                                    Required, but never shown







                                                    Required, but never shown

































                                                    Required, but never shown














                                                    Required, but never shown












                                                    Required, but never shown







                                                    Required, but never shown







                                                    Popular posts from this blog

                                                    Plaza Victoria

                                                    In PowerPoint, is there a keyboard shortcut for bulleted / numbered list?

                                                    How to put 3 figures in Latex with 2 figures side by side and 1 below these side by side images but in...